Art of Problem Solving

2025 AMC 10A Problems/Problem 16: Difference between revisions

Aoum (talk | contribs)
Blanked the page
Tag: Blanking
Boywithnuke (talk | contribs)
No edit summary
Line 1: Line 1:
==Question==
There are three jars. Each of three coins is placed in one of the tree jars, chosen at random and independently of the placement of the other coins. What is the expected number of coins in a jar with the most coins?


<imath>\textbf{(A) } \frac{4}{3}\qquad\textbf{(B) } \frac{13}{9}\qquad\textbf{(C) } \frac{5}{8}22\qquad\textbf{(D) } \frac{17}{9}\qquad\textbf{(E) } 2</imath>

Revision as of 12:32, 6 November 2025

Question

There are three jars. Each of three coins is placed in one of the tree jars, chosen at random and independently of the placement of the other coins. What is the expected number of coins in a jar with the most coins?

$\textbf{(A) } \frac{4}{3}\qquad\textbf{(B) } \frac{13}{9}\qquad\textbf{(C) } \frac{5}{8}22\qquad\textbf{(D) } \frac{17}{9}\qquad\textbf{(E) } 2$